Saltar al contenido principal
LibreTexts Español

6.9.E: Problemas en Maxima y Mínima

  • Page ID
    113763
  • \( \newcommand{\vecs}[1]{\overset { \scriptstyle \rightharpoonup} {\mathbf{#1}} } \) \( \newcommand{\vecd}[1]{\overset{-\!-\!\rightharpoonup}{\vphantom{a}\smash {#1}}} \)\(\newcommand{\id}{\mathrm{id}}\) \( \newcommand{\Span}{\mathrm{span}}\) \( \newcommand{\kernel}{\mathrm{null}\,}\) \( \newcommand{\range}{\mathrm{range}\,}\) \( \newcommand{\RealPart}{\mathrm{Re}}\) \( \newcommand{\ImaginaryPart}{\mathrm{Im}}\) \( \newcommand{\Argument}{\mathrm{Arg}}\) \( \newcommand{\norm}[1]{\| #1 \|}\) \( \newcommand{\inner}[2]{\langle #1, #2 \rangle}\) \( \newcommand{\Span}{\mathrm{span}}\) \(\newcommand{\id}{\mathrm{id}}\) \( \newcommand{\Span}{\mathrm{span}}\) \( \newcommand{\kernel}{\mathrm{null}\,}\) \( \newcommand{\range}{\mathrm{range}\,}\) \( \newcommand{\RealPart}{\mathrm{Re}}\) \( \newcommand{\ImaginaryPart}{\mathrm{Im}}\) \( \newcommand{\Argument}{\mathrm{Arg}}\) \( \newcommand{\norm}[1]{\| #1 \|}\) \( \newcommand{\inner}[2]{\langle #1, #2 \rangle}\) \( \newcommand{\Span}{\mathrm{span}}\)\(\newcommand{\AA}{\unicode[.8,0]{x212B}}\)

    Ejercicio\(\PageIndex{1}\)

    Verificar Nota 1.

    Ejercicio\(\PageIndex{1'}\)

    Completa los detalles faltantes en la prueba de los Teoremas 2 y 3.

    Ejercicio\(\PageIndex{2}\)

    Verificar los Ejemplos (A) y (B). Suplemento Ejemplo (A) aplicando el Teorema 2.

    Ejercicio\(\PageIndex{3}\)

    Prueba\(f\) para extremos en\(E^{2}\) si\(f(x, y)\) es
    (i)\(\frac{x^{2}}{2 p}+\frac{y^{2}}{2 q}(p>0, q>0)\);
    (ii)\(\frac{x^{2}}{2 p}-\frac{y^{2}}{2 q}(p>0, q>0)\);
    (iii)\(y^{2}+x^{4}\);
    (iv)\(y^{2}+x^{3}\).

    Ejercicio\(\PageIndex{4}\)

    (i) Encontrar el volumen máximo de un intervalo\(A \subset E^{3}\) (ver Capítulo 3, §7) cuyas longitudes de borde\(x, y, z\) tengan una suma prescrita:\(x+y+z=a\).
    (ii) Hacer lo mismo en\(E^{4}\) y en\(E^{n};\) espectáculo que\(A\) es un cubo.
    (iii) De ahí deducir que
    \[\sqrt[n]{x_{1} x_{2} \cdots x_{n}} \leq \frac{1}{n} \sum_{1}^{n} x_{k} \quad\left(x_{k} \geq 0\right),\]
    es decir, la media geométrica de los números\(n\) no negativos es\(\leq\) su media aritmética.

    Ejercicio\(\PageIndex{5}\)

    Encuentra el valor mínimo para la suma\(f(x, y, z, t)=x+y+z+t\) de cuatro números positivos con la condición de que\(x y z t=c^{4}\) (constante).
    [Respuesta:\(x=y=z=t=c; f_{\max }=4c\).]

    Ejercicio\(\PageIndex{6}\)

    Entre todos los triángulos inscritos en un círculo de radio\(R,\) encontramos el de área máxima.
    [Pista: Conecta los vértices con el centro. \(x, y, z\)Dejen ser los ángulos en el centro. Demostrar que el área del triángulo\(=\frac{1}{2} R^{2}(\sin x+\sin y+\sin z),\) con\(z=2 \pi-(x+y)\).]

    Ejercicio\(\PageIndex{7}\)

    Entre todos los intervalos\(A \subset E^{3}\) inscritos en el elipsoide se
    \[\frac{x^{2}}{a^{2}}+\frac{y^{2}}{b^{2}}+\frac{z^{2}}{c^{2}}=1\]
    encuentra el de mayor volumen.
    [Respuesta: las longitudes de los bordes son\(\frac{2 a}{\sqrt{3}}, \frac{2 b}{\sqrt{3}}, \frac{2 c}{\sqrt{3}}\).]

    Ejercicio\(\PageIndex{8}\)

    Dejar\(P_{i}=\left(a_{i} \cdot b_{i}\right), i=1,2,3,\) ser 3 puntos en la\(E^{2}\) formación de un triángulo en el que un ángulo (digamos,\(\,easureangle P_{1})\) es\(\geq 2 \pi / 3\).
    Encontrar un punto\(P=(x, y)\) para el que la suma de las distancias,
    \[P P_{1}+P P_{2}+P P_{3}=\sum_{i=1}^{3} \sqrt{\left(x-a_{i}\right)^{2}+\left(y-b_{i}\right)^{2}},\]
    sea lo menos posible.
    [Esquema: Vamos\(f(x, y)=\sum_{i=1}^{3} \sqrt{\left(x-a_{i}\right)^{2}+\left(y-b_{i}\right)^{2}}\).
    Demostrar que no\(f\) tiene derivadas parciales en\(P_{1}, P_{2},\) o\(P_{3}\) (\(P_{1}, P_{2},\)y así y\(P_{3}\) son puntos críticos en los que puede ocurrir un extremo), mientras que en otros puntos\(P,\) los parciales existen pero nunca desaparecen simultáneamente, de manera que no hay otros críticos puntos.
    En efecto, probar eso\(D_{1} f(P)=0=D_{2} f(P)\) implicaría que
    \[\sum_{i=1}^{3} \cos \theta_{i}=0=\sum_{1}^{3} \sin \theta_{I},\]
    dónde\(\theta_{i}\) está el ángulo entre\(\overline{P P_{i}}\) y el\(x\) -eje; de ahí
    \[\sin \left(\theta_{1}-\theta_{2}\right)=\sin \left(\theta_{2}-\theta_{3}\right)=\sin \left(\theta_{3}-\theta_{1}\right) \quad\text {(why?),}\]
    y tan\(\theta_{1}-\theta_{2}=\theta_{2}-\theta_{3}=\theta_{3}-\theta_{1}=2 \pi / 3,\) contrario a\(\angle P_{1} \geq 2 \pi / 3.\) (¿Por qué?)
    A partir de consideraciones geométricas, concluimos que\(f\) tiene un mínimo absoluto en\(P_{1}\).
    (Esto demuestra que no se pueden ignorar los puntos en los que no\(f\) tiene parciales.)]

    Ejercicio\(\PageIndex{9}\)

    Continuando Problema 8, mostrar que si ninguno de\(\angle P_{1}, \angle P_{2},\) y\(\angle P_{3}\) es\(\geq\)\(2 \pi / 3,\) entonces\(f\) alcanza su menor valor en algunos\(P\) (dentro del triángulo) tal que\(\angle P_{1} P P_{2}=\angle P_{2} P P_{3}=\angle P_{3} P P_{1}=2 \pi / 3\).
    [Pista: Verifíquelo\(D_{1} f=0=D_{2} f\) en\(P\).
    Utilizar la ley de los cosenos para demostrar eso\(P_{1} P_{2}>P P_{2}+\frac{1}{2} P P_{1}\) y\(P_{1} P_{3}>P P_{3}+\frac{1}{2} P P_{1}\).
    Sumando, obtener\(P_{1} P_{3}+P_{1} P_{2}>P P_{1}+P P_{2}+P P_{3},\) i.e.,\(f\left(P_{1}\right)>f(P).\) Similarmente,\(f\left(P_{2}\right)>f(P)\) y\(f\left(P_{3}\right)>f(P).\)
    Combinando con el Problema 8, obtener el resultado.]

    Ejercicio\(\PageIndex{10}\)

    En un círculo de radio\(R\) inscribe un polígono con\(n+1\) lados de área máxima.
    [Contorno: Dejen\(x_{1}, x_{2}, \ldots, x_{n+1}\) ser los ángulos centrales subtendidos por los lados del polígono. Entonces su área\(A\) es
    \[\frac{1}{2} R^{2} \sum_{k=1}^{n+1} \sin x_{k},\]
    con\(x_{n+1}=2 \pi-\sum_{k=1}^{n} x_{k}.\) (¿Por qué?) Así todo se reduce a maximizar con
    \[f\left(x_{1}, \ldots, x_{n}\right)=\sum_{k=1}^{n} \sin x_{k}+\sin \left(2 \pi-\sum_{k=1}^{r_{k}} x_{k}\right),\]
    la condición de que\(0 \leq x_{k}\) y\(\sum_{k=1}^{n} x_{k} \leq 2 \pi.\) (¿Por qué?)
    Estas desigualdades definen un conjunto acotado\(D \subset E^{n}\) (llamado simplex). Equiparando todos los parciales de\(f\) para\(0,\) mostrar que el único punto crítico interior a\(D\) es\(\vec{x}=\left(x_{1}, \ldots, x_{n}\right),\) con\(x_{k}=\frac{2 \pi}{n+1}, k \leq n\) (lo que implica eso\(x_{n+1}=\frac{2 \pi}{n+1},\) también). Para eso\(\vec{x},\) obtenemos
    \[f(\vec{x})=(n+1) \sin [2 \pi /(n+1)].\]
    Este valor debe compararse con los valores de “límite” de\(f,\) en las “caras” del simplex D (ver Nota 4).
    Haz esto por inducción. Para el\(n=2,\) Problema 6 muestra que efectivamente\(f(\vec{x})\) es el más grande cuando todos\(x_{k}\) iguales\(\frac{2 \pi}{n+1}.\) Ahora deja\(D_{n}\) ser la “cara” de\(D,\) donde\(x_{n}=0.\) En esa cara, tratar\(f\) como una función de solo\(n-1\) variables,\(x_{1}, \ldots, x_{n-1}\).
    Por la hipótesis inductiva, el mayor valor de\(f\) on\(D_{n}\) es\(n \sin (2 \pi / n).\) Similarmente para las otras “caras”. A medida\(n \sin (2 \pi / n)<(n+1) \sin 2 \pi /(n+1),\) que la inducción es completa.
    Así, el área\(A\) es la más grande cuando el polígono es regular, para lo cual
    \[\left.A=\frac{1}{2} R^{2}(n+1) \sin \frac{2 \pi}{n+1}.\right]\]

    Ejercicio\(\PageIndex{11}\)

    Entre todos los triángulos de un perímetro prescrito\(2p,\) encontramos el de área máxima.
    [Pista: Maximizar con\(p(p-x)(p-y)(p-z)\) la condición de que\(x+y+z=2 p\).]

    Ejercicio\(\PageIndex{12}\)

    Entre todos los triángulos de área\(A,\) encontramos el de perímetro más pequeño.

    Ejercicio\(\PageIndex{13}\)

    Encuentra la distancia más corta desde un punto dado\(\vec{p} \in E^{n}\) a un plano determinado\(\vec{u} \cdot \vec{x}=c\) (Capítulo 3, §§4-6). Respuesta:
    \[\pm \frac{\vec{u} \cdot \vec{p}-c}{|\vec{u}|}.\]
    [Pista: Primero hazlo\((x, y, z)\) por\(E^{3},\) escrito para\(\vec{x}\).]


    6.9.E: Problemas en Maxima y Mínima is shared under a CC BY 1.0 license and was authored, remixed, and/or curated by LibreTexts.